Fixed typo in Young_and_Freedman_12/problem26.91.tex
[course.git] / latex / problems / Young_and_Freedman_12 / problem26.91.tex
index 5e74a0f61049fd998d5388dd71acd34047c27788..157e691b2045563091a5648f11c7db1353a851b1 100644 (file)
@@ -5,7 +5,7 @@ resistance $R_T$ of the infinite network is equal to
 \begin{equation}
   R_T = R_1 + \sqrt{R_1^2 + 2R_1R_2}
 \end{equation}
-(\emph{Hint:} Since the network is infinite, the sestance of the
+(\emph{Hint:} Since the network is infinite, the resistance of the
 network to the right of points $c$ and $d$ is also equal to $R_T$.)
 \begin{center}
 \begin{verbatim}